5
$\begingroup$

This was post by me on Maths SE: but it did not get any solution.

Some months ago I made the following conjecture -
Let $d(n)$ denote the number of divisors of $n $.
Then let $N$ be a number such that $d(N)$ divides $N$ . Also let $I= \frac{N}{d(N)}$ which is defined as the "Index of Beauty of $N$ ". Then, For every number $I$ there exists a number $N$ such that $I$ is the index of beauty of $N$.

This conjecture was proved false by Greg Martin here.
He said that it can be showed by exaustive computation that the following $I$ fail the conjecture under $1000$ are $\{18, 27, 30, 45, 63, 64, 72, 99, 105, 112, 117, 144, 153, 160, 162, 165, 171, 195, 207, 225, 243, 252, 255, 261, 279, 285, 288, 294, 320, 333, 336, 345, 352, 360, 369, 387, 396, 405, 416, 423, 435, 441, 465, 468, 477, 490, 504, 531, 544, 549, 555, 567, 576, 603, 608, 612, 615, 616, 625, 639, 645, 657, 684, 705, 711, 726, 728, 735, 736, 747, 792, 795, 801, 810, 828, 840, 873, 880, 885, 891, 909, 915, 927, 928, 936, 952, 960, 963, 981, 992\}$

Now what I am interested is that sequence of $I$ that fails.
(i) Is this sequence infinite?How?
(ii)Is there any approximation which can tell the number of such failed $I$ less than a fixed $x$
(iii)If the sequence is infinite then are there canonical forms in which all of the values are in our list.

$\endgroup$
5
  • 2
    $\begingroup$ This question is too basic for mathoverflow. You may find oeis.org/A036763 useful, though. $\endgroup$
    – Charles
    Jun 2, 2014 at 18:31
  • 5
    $\begingroup$ I disagree - I think this is unlikely to be resolved except through research-level methods, yet it is quite possible that it can be resolved, at least partially. So I think the question fits the parameters for mathoverflow. $\endgroup$ Jun 2, 2014 at 18:37
  • 2
    $\begingroup$ This questions is not too basic for mathoverflow. How can it be so when I can barely even understand the question? $\endgroup$
    – user51538
    Jun 2, 2014 at 18:57
  • $\begingroup$ @GregMartin: In that case I'll answer it. $\endgroup$
    – Charles
    Jun 2, 2014 at 19:15
  • 1
    $\begingroup$ Since this question now has 4 close votes, I wish to say that I agree with Greg Martin that the question seems perfectly fine for MO. I don't know how many exceptional $I$ there are up to $x$, and it seems not easy. Naturally there may be many opinions on the interest of the question, but perhaps that applies to many problems on MO. $\endgroup$
    – Lucia
    Jun 2, 2014 at 23:50

2 Answers 2

16
$\begingroup$

Here's an elementary argument proving that the set of numbers $I$ that fail the conjecture is infinite.

Claim. $p^{17} \in I$ for all primes $p > 19$.

Proof. Suppose that $p^{17}=\frac{N}{d(N)}$ for some $N$. Write $N=p^{17+k}n$ where $p$ does not divide $n$. Then, $d(N)=(18+k)d(n)$, and so $p^kn=(18+k)d(n)$. Since $p>19$ and $n \geq d(n)$ this can only hold if $k=0$. But now $\frac{n}{d(n)}=18$, which Greg Martin has shown is impossible.

$\endgroup$
5
  • 5
    $\begingroup$ Here is a variant of your construction which produces many excluded numbers. I claim that if $p$ (a prime) is large then $15p$ fails the conjecture. For if $n=15pd(n)$ and $p$ is large then $p$ can divide $n$ only to the power $1$ (use $d(n)\le n^{1/3}$ for large $n$). If now $n=pm$ with $(p,m)=1$ then we must have $m=30d(m)$, contradicting Greg Martin's result that $30$ is excluded. In fact, inspecting his table it looks like $15p$ is excluded for all primes $p>5$. $\endgroup$
    – Lucia
    Jun 3, 2014 at 2:16
  • $\begingroup$ @Lucia extremely good proof $\endgroup$ Jun 3, 2014 at 2:25
  • $\begingroup$ @Lucia can you prove that if the number in the list is in the form $15n$ then $n$ is a prime. $\endgroup$ Jun 3, 2014 at 2:26
  • 1
    $\begingroup$ @ShivamPatel: Not true -- $810$, $840$, $960$ etc. I'm afraid I don't know any more about this sequence! $\endgroup$
    – Lucia
    Jun 3, 2014 at 2:28
  • $\begingroup$ @Lucia sorry ..I did not see the list ...just as a curiosity so can you tell me the distribution of numbers in form $10n$ in the sequence $\endgroup$ Jun 3, 2014 at 2:30
2
$\begingroup$

This is a simple consequence of the density of refactorable numbers, http://oeis.org/A033950, and the growth of the divisor function. Basically, $n/d(n)>x$ if $x>n^{1+\frac{0.7}{\log\log n}}$ (where the constant could be anything greater than $\log 2$) with only finitely many exceptions. But of these numbers only

$$ O\left(\frac{(\log\log x)^{k^3-1}}{\log x}\right)=\\ O\left(\frac{(\log(\log n+\frac{0.7\log n}{\log\log n}))^{k^3-1}}{\log n+\frac{0.7\log n}{\log\log n}}\right)=\\ O\left(\frac{(\log\log n)^{k^3-1}}{\log n}\right) $$

are refactorable (for every $k>1$), so there must be infinitely many exceptions. Further, these 'exceptions' have density 1 (though the constants are nasty).

$\endgroup$
7
  • $\begingroup$ Hmm. n/ something bigger than 1 is bigger than x if x is bigger than n times something else bigger than 1. One of us is doing something wrong. $\endgroup$ Jun 2, 2014 at 19:38
  • $\begingroup$ I'm not sure I understand your argument. What exactly are you claiming about the set of numbers of the form $n/d(n)$? $\endgroup$
    – Lucia
    Jun 2, 2014 at 19:44
  • $\begingroup$ All the $+$ should be $-$. $\endgroup$ Jun 2, 2014 at 22:55
  • $\begingroup$ And the second inequality needs to read the other way round, that is, $x<n^{1-0.7/\log\log n}$. $\endgroup$ Jun 2, 2014 at 23:01
  • 3
    $\begingroup$ @Charles: I think your argument is unclear and possibly flawed -- at least I haven't understood it. Could you write it carefully -- it has several typos, and the second part of it seems unclear to me. $\endgroup$
    – Lucia
    Jun 2, 2014 at 23:28

Your Answer

By clicking “Post Your Answer”, you agree to our terms of service and acknowledge you have read our privacy policy.

Not the answer you're looking for? Browse other questions tagged or ask your own question.